- PowerScore Staff
- Posts: 5972
- Joined: Mar 25, 2011
- Mon Jul 11, 2016 11:51 am
#26966
Complete Question Explanation
The correct answer choice is (A)
This problem can be solved by looking solely at seat 2:
Answer choice (B) is incorrect because O’s seat is in the last row.
Answer choice (C) is incorrect because we established in the setup that P’s seat was seat 5.
Answer choice (D) is incorrect because R’s seat cannot be in the first row (R must sit in seat 4 or 6).
Answer choice (E) is incorrect because U cannot be assigned to seat 2.
Accordingly, answer choice (A) is proven correct by process of elimination.
The correct answer choice is (A)
This problem can be solved by looking solely at seat 2:
Answer choice (B) is incorrect because O’s seat is in the last row.
Answer choice (C) is incorrect because we established in the setup that P’s seat was seat 5.
Answer choice (D) is incorrect because R’s seat cannot be in the first row (R must sit in seat 4 or 6).
Answer choice (E) is incorrect because U cannot be assigned to seat 2.
Accordingly, answer choice (A) is proven correct by process of elimination.
Dave Killoran
PowerScore Test Preparation
Follow me on X/Twitter at http://twitter.com/DaveKilloran
My LSAT Articles: http://blog.powerscore.com/lsat/author/dave-killoran
PowerScore Podcast: http://www.powerscore.com/lsat/podcast/
PowerScore Test Preparation
Follow me on X/Twitter at http://twitter.com/DaveKilloran
My LSAT Articles: http://blog.powerscore.com/lsat/author/dave-killoran
PowerScore Podcast: http://www.powerscore.com/lsat/podcast/